Đến nội dung

hoangquochung3042002 nội dung

Có 175 mục bởi hoangquochung3042002 (Tìm giới hạn từ 25-05-2020)



Sắp theo                Sắp xếp  

#667486 Tìm giá trị Min của: $P=x^3+y^3$

Đã gửi bởi hoangquochung3042002 on 07-01-2017 - 18:30 trong Bất đẳng thức và cực trị

1.Cho:$\frac{a-1}{2}=\frac{b+3}{4}=\frac{c-5}{6}$ và 5a-4c-3b=46. Tìm a,b,c.

2. Rút gọn biểu thức:

A=$\frac{1}{\sqrt{1}+\sqrt{2}}+\frac{1}{\sqrt{2}+\sqrt{3}}+\frac{1}{\sqrt{3}+\sqrt{4}}$+...+$\frac{1}{\sqrt{n-1}+\sqrt{n}}$

Ta có: $\frac{a-1}{2}=\frac{b+3}{4}=\frac{c-5}{6}<=>\frac{5a-5}{10}=\frac{3b+9}{12}=\frac{4c-20}{24}$

Áp dụng tinh chat day ti so bang nhau:$\frac{5a-5}{10}=\frac{3b+9}{12}=\frac{4c-20}{24}=\frac{5a-3b-4c-5-9+20}{10-12-24}=-2$

=> a=-3;b=-11;c=-7.




#667502 Tìm giá trị Min của: $P=x^3+y^3$

Đã gửi bởi hoangquochung3042002 on 07-01-2017 - 20:22 trong Bất đẳng thức và cực trị

Tìm nghiệm nguyên dương của phương trình:

x2-6xy+13y2=100

Ta có: $x^2-6xy+13y^2=100<=>(x-3y)^2+4y^2=100=0+100=100+0=36+64=64+36.$

Vì x,y nguyên dương

=>TH1:x=15;y=5;

    TH2:x=10;y=0;

    TH3:x=18;y=4;

    TH4:x=17;y=3.




#667626 Tìm giá trị Min của: $P=x^3+y^3$

Đã gửi bởi hoangquochung3042002 on 08-01-2017 - 16:17 trong Bất đẳng thức và cực trị

Tìm nghiệm nguyên dương của phương trình:5x+7y=112

Ta có: $5x+7y=112<=>x=\frac{112-7y}{5}.$ 

x nguyên dương =>$7y<112;(112-7y)$ chia hết cho 5.

mà y nguyên dương => hoặc y=1 hoặc y=11.

 y=1=>x=21;

 y=11=>x=7.




#667769 Giải phương trình

Đã gửi bởi hoangquochung3042002 on 09-01-2017 - 19:52 trong Phương trình - hệ phương trình - bất phương trình

$\sqrt{2x^2-2x+5}+1-x=\sqrt{x+2}$

ĐK: ($\frac{3-\sqrt{13}}{2}\leq x\leq \frac{3+\sqrt{13}}{2}$).
$PT<=>\sqrt{2x^2-2x+5}=\sqrt{x+2}+x-1<=>2x^2-2x+5=x^2-x+3-2(x-1)\sqrt{x+2}<=>x^2-x+2=-2(x-1)\sqrt{x+2}<=>x^4+x^2+4-2x^3-4x+4x^2=(4x^2-8x+4)(x+2)<=>x^4-6x^3+5x^2+8x-4<=>(x-2)(x+1)(2x-5-\sqrt{17})(2x-5+\sqrt{17})=0.$
Kết hợp ĐK:=> hoặc x=2 hoặc x=-1 hoặc x=$\frac{5+\sqrt{17}}{2}$.



#667772 Tìm x,y,z biết $x^2+9y^2+z^2=-2x+12y+4z-9$

Đã gửi bởi hoangquochung3042002 on 09-01-2017 - 20:04 trong Đại số

Tìm x,y,z biết $x^2+9y^2+z^2=-2x+12y+4z-9$

$PT<=>(x^2+2x+1)+(9y^2-12y+4)+(z^2-4z+4)=0<=>(x+1)^2+(3y-2)^2+(z-2)^2=0.$

=> x=-1; y=$\frac{2}{3}$; z=2.




#667793 Giải phương trình

Đã gửi bởi hoangquochung3042002 on 09-01-2017 - 21:01 trong Phương trình - hệ phương trình - bất phương trình

Cảm ơn bạn. Cách của mình, cũng tương tự   :lol:

ĐK: $x\geq -2$

pt $\Leftrightarrow \sqrt{2x^2-2x+5}=x-1+\sqrt{x+2}(1)\Rightarrow x^2-x+2=2(x-1)\sqrt{x+2}=0$

Đặt $\sqrt{x+2}=t\geq 0\Leftrightarrow x=t^2-2\Leftrightarrow x^2-x+2-2(x-1)t=0\Leftrightarrow x^2+t^2-2-2x+2-2xt+2t=0\Leftrightarrow t^2+2t(1-x)+x^2-2x=0. \Delta '=1$

$\Rightarrow$ t=x hoặc t=x-2

Bài này là bài dê. Có một số bài phải biến đổi cực kì phức tạp.




#668135 Giải phương trình

Đã gửi bởi hoangquochung3042002 on 13-01-2017 - 11:52 trong Phương trình - hệ phương trình - bất phương trình

Làm giùm em bài này với

Bai nao v. Neu duoc anh se giup.




#668476 Tìm x,y€Z

Đã gửi bởi hoangquochung3042002 on 15-01-2017 - 21:17 trong Toán rời rạc

Xy—4y+x=12

Ta có:

xy-4y+x=12<=>y(x-4)+(x-4)=8<=>(x-4)(y+1)=8=$\pm$1.$\pm$8=$\pm 8.\pm 1$=$\pm 2.\pm 4$=$\pm 4.\pm 2$.

Vì x,y nguyên=>các trường hợp.

TH1:x=5;y=7;

TH2:x=3;y=-9;

TH3:x=12;y=0;

TH4:x=-4;y=-2;

TH5:x=6;y=8;

TH6:x=2;y=-5;

TH7:x=8;y=1;

TH8:x=0;y=-3.




#668586 Tính tổng S

Đã gửi bởi hoangquochung3042002 on 16-01-2017 - 21:24 trong Đại số

Tính tổng $S=\sqrt{1+\frac{1}{1^2}+\frac{1}{2^2}}+\sqrt{1+\frac{1}{2^2}+\frac{1}{3^2}}+...+\sqrt{1+\frac{1}{2015^2}+\frac{1}{20116^2}}$

Ta có: $1+\frac{1}{n^2}+\frac{1}{(n+1)^2}=\frac{(n^2+n)^2+n^2+2n+1+n^2}{n^2(n+1)^2}=\frac{(n^2+n)^2+2(n^2+n)+1}{n^2(n+1)^2}=\frac{(n^2+n+1)^2}{n^2(n+1)^2}.$

$=>\sqrt{1+\frac{1}{n^2}+\frac{1}{(n+1)^2}}=\frac{n^2+n+1}{n(n+1)}=1+\frac{1}{n}-\frac{1}{n+1}.$

Sau đó bạn áp dụng vào là được nhé.




#668715 CM: $x^4+x+2>0$.

Đã gửi bởi hoangquochung3042002 on 17-01-2017 - 21:16 trong Bất đẳng thức và cực trị

Chứng minh BĐT sau: $x^4+x+2>0$.

$x^4+x+2=(x^2-\frac{1}{2})^2+x^2+x+\frac{7}{4}>0.$




#669126 Chứng minh giá trị nhỏ nhất của P bằng 0

Đã gửi bởi hoangquochung3042002 on 20-01-2017 - 22:01 trong Đại số

Cho biểu thức $P=x^{2}+xy+y^{2}-3(x+y)+3$. Chứng minh rằng giá trị nhỏ nhất của P bằng 0

$P=(x+\frac{y-3}{2})^2+\frac{3(y-1)^2}{4}\geq 0.$ 

Dấu "=" xảy ra khi x=y=1.




#669132 Chứng minh giá trị nhỏ nhất của P bằng 0

Đã gửi bởi hoangquochung3042002 on 20-01-2017 - 22:08 trong Đại số

Bạn giải ra chi tiết được không 

$P=x^2+x(y-3)+y^2-3y+3=(x^2+\frac{x(y-3)}{2}+\frac{(y-3)^2}{4})-\frac{(y-3)^2}{4}+y^2-3y+3=(x+\frac{y-3}{2})^2+\frac{3(y-1)^2}{4} \geq 0$.




#669137 min $M=a^{2}+ab+b^{2}-3a-3b+2001$

Đã gửi bởi hoangquochung3042002 on 20-01-2017 - 22:29 trong Đại số

Cho biểu thức $M=a^{2}+ab+b^{2}-3a-3b+2001$. Với giá trị nào của a và b thì M đạt giá trị nhỏ nhất? Tìm giá trị nhỏ nhất đó

$M=(a+\frac{b-3}{2})^2+\frac{3(b-1)^2}{4}+1998\geq 1998.$

Dấu "=" xảy ra khi a=b=1.




#669178 Tìm GTNN của $\frac{x^{4}+4x^{3}+4x^{...

Đã gửi bởi hoangquochung3042002 on 21-01-2017 - 13:11 trong Bất đẳng thức và cực trị

Tìm GTNN của $\frac{x^{4}+4x^{3}+4x^{2}+9}{x^{2}+2x}$ với x>0

$\frac{x^4+4x^3+4x^2+9}{x^2+2x}=x^2+2x+\frac{9}{x^2+2x}\geq 6.$

Dấu "=" xảy ra khi x=1.

Bài này ban tach phan tren la dc.




#669231 Toán violympic

Đã gửi bởi hoangquochung3042002 on 21-01-2017 - 20:53 trong Đại số

Trong mặt phẳng Oxy cho điểm A di động trên y=x^2.Tập hợp trung điểm I của đoạn OA là parabol.

Để đa thức P(x)=ax^4+bx^3+1 chia cho Q(x)=(x-1)^2 thì điều kiện là:

a) a=3,b=-4  b)a=-3,b=-4   c) a=-3,b=4    d)a=3,b=4

P(1)=a+b+1=0=>a+b=-1;

ĐÁP ÁN a).




#669347 Toán violympic

Đã gửi bởi hoangquochung3042002 on 22-01-2017 - 15:02 trong Đại số

cách làm chi tiết tí đi. À làm hộ câu 1 nữa nhé !!!

thực hiện phép chia đa thức thông thường, ta có: 4a+3b=0.

kết hợp với p(1)=0=> a+b=-1 =>a=3;b=-4.




#669356 Giải hệ: $\sqrt{x}+\sqrt{2012-y}=\sq...

Đã gửi bởi hoangquochung3042002 on 22-01-2017 - 15:15 trong Phương trình, hệ phương trình và bất phương trình

Giải hệ: 
$\sqrt{x}+\sqrt{2012-y}=\sqrt{2012} $
$\sqrt{y}+\sqrt{2012-x}=\sqrt{2012}$

lấy vế trên trừ vế dưới sau đó nhân liên hợp ta được: $(x-y)(\frac{1}{\sqrt{x}+\sqrt{y}}+\frac{1}{\sqrt{2012-x}+\sqrt{2012-y}})=0. =>x=y.$

rồi từ đay dễ r.




#669400 $a)\sqrt{26}+\sqrt{8}>\sqrt{48}$

Đã gửi bởi hoangquochung3042002 on 22-01-2017 - 18:54 trong Đại số

Chứng minh các bất đẳng thức sau:

$a)\sqrt{26}+\sqrt{8}>\sqrt{48}$ 

$b)\frac{5+\sqrt{5}}{5-\sqrt{5}}+\frac{5-\sqrt{5}}{5+\sqrt{5}}-\sqrt{10}<0$

$c)(\frac{\sqrt{5}+1}{1+\sqrt{5}+\sqrt{3}}+\frac{\sqrt{5}-1}{1+\sqrt{3}-\sqrt{5}})(\sqrt{3}-4\sqrt{\frac{1}{3}}+2)\sqrt{0.2}-\sqrt{1.01}>0$ 

$d)\frac{\sqrt{2}+\sqrt{3}-1}{2+\sqrt{6}}+\frac{\sqrt{2}-\sqrt{3}}{2\sqrt{6}}(\frac{\sqrt{3}}{2-\sqrt{6}}+\frac{\sqrt{3}}{2+\sqrt{6}})-\frac{1}{\sqrt{2}}+\sqrt{3-\sqrt{2}}>0$ $e)\sqrt{\sqrt{2}+2\sqrt{\sqrt{2}-1}}+\sqrt{\sqrt{2}-2\sqrt{\sqrt{2}-1}}>1,9$ 

$g)\sqrt{\sqrt{17+12\sqrt{2}}-\sqrt{2}}>\sqrt{3}-1$ 

$h)(\sqrt{\sqrt{3}}+\sqrt{\sqrt{5}}+\sqrt{\sqrt{7}})-(\sqrt{3}+\sqrt{5}+\sqrt{7})<3$ $i)\frac{\sqrt{\sqrt{2}+\sqrt{2}}+\sqrt{3}\sqrt{2-\sqrt{2}}}{4}<0,8$

câu g)

$\sqrt{\sqrt{17+12\sqrt{2}}-\sqrt{2}}>\sqrt{3}-1<=>\sqrt{3+2\sqrt{2}-\sqrt{2}}>\sqrt{3}-1<=>\sqrt{3+\sqrt{2}}>\sqrt{3}-1$

Bình phương hai vế :$<=>3+\sqrt{2}>4-2\sqrt{3}<=>\sqrt{2}+2\sqrt{3}-1>0.$(luôn đúng)=> BĐT cần chứng minh đúng.

câu h)

Dễ thấy: $\sqrt{\sqrt{3}}<\sqrt{3}$; $\sqrt{\sqrt{5}}<\sqrt{5}$; $\sqrt{\sqrt{7}}<\sqrt{7}$=> VT<0<3.

câu i)

Dễ chứng minh: $\sqrt{\sqrt{2}+\sqrt{2}}<1.7$; $\sqrt{3}\sqrt{2-\sqrt{2}}<1.5$ =>$\frac{\sqrt{\sqrt{2}+\sqrt{2}}+\sqrt{3}\sqrt{2-\sqrt{2}}}{4}<0.8$.




#669401 Điều kiện bình phương 2 vế

Đã gửi bởi hoangquochung3042002 on 22-01-2017 - 19:00 trong Đại số

Tại sao khi muốn bình phương 2 vế thì 2 vế phải đều không âm?

Đây là 1 kiến thức cũng khá căn bản trong khi giải các bài toán đại số, nhưng mình vẫn chưa thật sự hiểu rõ nguyên nhân. Mong các bạn giải thích giúp mình.
Cám ơn các bạn.

Bình phương hai vế áp dụng cho BĐT.

Có thể lấy ví dụ de hieu nguyen nhan vi sao can binh phuong hai ve.

-2<0 thì nếu bih phuogn 2 vế lên thì ko dc BĐT moi tuog duong.




#669472 $a)\sqrt{26}+\sqrt{8}>\sqrt{48}$

Đã gửi bởi hoangquochung3042002 on 22-01-2017 - 22:22 trong Đại số

Câu i) bạn bấm máy à sao biết bé hơn 1,5

nhân căn 3 vào, bình phuong lien tuc là dc.




#669660 Tại sao $\frac{1}{2}$ không phải là nghiệm...

Đã gửi bởi hoangquochung3042002 on 24-01-2017 - 09:50 trong Đại số

Hi mọi người,

 

Mình có bài tập như sau: Giải phương trình: $(8x + 5)^{2}(4x + 3)(2x + 1) = 9$

 

Còn đây là bài giải của mình:

 

Đặt $2x + 1 = a$. Ta có:

$(4a + 1)^{2}(2a + 1)a = 9$

$\Leftrightarrow (16a^{2} + 8a + 1)(2a + 1)a = 9$

$\Leftrightarrow a(32a^{3} + 32a^{2} + 10a + 1) = 9$

$\Leftrightarrow 32a^{4} + 32a^{3} + 10a^{2} + a - 9 = 0$

$\Leftrightarrow 32a^{3}(a + 1) + 10a(a + 1) - 9(a + 1) = 0$

$\Leftrightarrow (a + 1)(32a^{3} + 10a - 9) = 0$

$\Leftrightarrow (a + 1)(32a^{3} - 16a^{2} + 16a^{2} - 8a + 18a - 9) = 0$

$\Leftrightarrow (a + 1)[16a^{2}(2a - 1) + 8a(2a - 1) + 9(2a - 1)] = 0$

$\Leftrightarrow (a + 1)(2a - 1)(16a^{2} + 8a + 9) = 0$

Vì $16a^{2} + 8a + 9 = (4a + 1)^{2} + 8 > 0  \forall  a$ nên:

$a + 1 = 0 \Leftrightarrow a = -1$

$2a - 1 = 0 \Leftrightarrow a = \frac{1}{2}$

 

Đó là bài giải của mình. Khi mình thay $-1$ vào phương trình ban đầu thì đúng. Nhưng nếu mình thay $\frac{1}{2}$ vào phương trình ban đầu thì kết quả sai. Vậy câu hỏi của mình là tại sao mình giải đúng mà kết quả $\frac{1}{2}$ lại không phải là nghiệm ?

 

Lưu ý: Mình cần mọi người chỉ ra chỗ sai trong bài giải của mình đã nhé. Còn những cách giải hay hơn thì có thể nói sau ^^

 

Mình cảm ơn.

 

 

Edited: À quên mất, kết quả mình tìm được là $a$ chứ không phải $x$ nên thế vào nó sai. Topic đã giải quyết (sau 5s kể từ khi post bài ^^). Nhờ Administrators & Moderators xóa hộ!

vãi. thay a vào chô 2x+1=a mới tìm ra x rồi mới dc thay vào p/trinh.

a=-1=> x=-1;

a=$\frac{1}{2}$=> x=$\frac{-1}{4}.




#669736 Tìm tích xy

Đã gửi bởi hoangquochung3042002 on 24-01-2017 - 20:04 trong Phương trình, hệ phương trình và bất phương trình

Cho x,y là các số thỏa mãn hệ phương trình $\left\{\begin{matrix} x^2+ & \frac{1}{y^2} &+\frac{x}{y} =3\\ x & +\frac{1}{y} & +\frac{x}{y}=3 \end{matrix}\right.$ . Tìm tích xy

Cộng p/trinh trên cho phương trình dưới ta được: $(x+\frac{1}{y})^2+(x+\frac{1}{y})=6.$

Đặt: $a=x+\frac{1}{y}.$=> $a^2+a-6=0$=> a=2 hoặc a=-3.

Tới đây thì dễ rồi.




#669742 $\frac{a}{a^2+1}+\frac{b}{b^2+1}+\frac{c}{c^2+1}\leq...

Đã gửi bởi hoangquochung3042002 on 24-01-2017 - 20:28 trong Bất đẳng thức và cực trị

nếu $a,b,c\geq 0 ; a+b+c = 1 thì  M=\frac{a}{a^2+1}+\frac{b}{b^2+1}+\frac{c}{c^2+1}\leq \frac{9}{10}$

CM tương đương.

chuyển vế: $(\frac{a}{a^2+1}-\frac{3}{10})+(\frac{b}{b^2+1}-\frac{3}{10})+(\frac{c}{c^2+1}-\frac{3}{10})\leq 0<=>\sum( \frac{-3a^2+10a-3}{a^2+1})$(luôn đúng).

Dấu "=" xảy ra khi x=y=3.




#669746 $\frac{a}{a^2+1}+\frac{b}{b^2+1}+\frac{c}{c^2+1}\leq...

Đã gửi bởi hoangquochung3042002 on 24-01-2017 - 20:36 trong Bất đẳng thức và cực trị

x=y=1/3 chứ

a=b=c=3 mới đúng thay vào thử ik bn.




#669839 CMR: n2+3n+5 không chia hết cho 121 với n là số tự nhiên

Đã gửi bởi hoangquochung3042002 on 25-01-2017 - 11:00 trong Đại số

CMR: n2+3n+5 không chia hết cho 121 với n là số tự nhiên

Gỉa sử tồn tại số tự nhiên n thỏa $n^2+3n+5$$\vdots$121.

=>$4(n^2+3n+5) \vdots 121<=>[(2n+3)^2+11]\vdots 121$.

Mặt khác, $n^2+3n+5$ $\vdots$ 11 (vì chia hết cho 121) => (2n+3)^2$\vdots$ 11.

mà 11 là số tự nhiên nguyên tố nên (2n+3)^2 $\vdots$ 121

=> (2n+3)^2+11  ko chia hết cho 121

=>dpcm.